Which of the following conditions / properties ensures that the $ntimes n$ matrix $A$ is invertible?












1












$begingroup$



Question: Which of the following conditions / properties ensure that the $ntimes n$ matrix $A$ is invertible?



Possible answers:





  1. $A^Tcdot A$ is invertible

  2. $A^T= A$

  3. $A^2+ A + I = 0$


  4. $AB=BA$, for any invertible matrix $B$




My Answer:
My answer is that number $1$ and $4$ are conditions that ensure that $ntimes n$ matrix $A$ is invertible. Am I correct?










share|cite|improve this question











$endgroup$








  • 1




    $begingroup$
    @Suraj You should probably give your response in an answer instead of comments
    $endgroup$
    – Shubham Johri
    Dec 19 '18 at 17:04










  • $begingroup$
    so 3 is the only conditions that ensures that n*n matrix A is invertible
    $endgroup$
    – anders
    Dec 19 '18 at 17:04












  • $begingroup$
    @anders Both $(1),(3)$ ensure the invertibility of $A$
    $endgroup$
    – Shubham Johri
    Dec 19 '18 at 17:05










  • $begingroup$
    oh okey. thank you so much
    $endgroup$
    – anders
    Dec 19 '18 at 17:05










  • $begingroup$
    $(1)$ is true, as $det(A^TA)=det(A^T)cdotdet(A)=det(A)^2ne0impliesdet(A)ne0$.
    $endgroup$
    – Shubham Johri
    Dec 19 '18 at 17:06
















1












$begingroup$



Question: Which of the following conditions / properties ensure that the $ntimes n$ matrix $A$ is invertible?



Possible answers:





  1. $A^Tcdot A$ is invertible

  2. $A^T= A$

  3. $A^2+ A + I = 0$


  4. $AB=BA$, for any invertible matrix $B$




My Answer:
My answer is that number $1$ and $4$ are conditions that ensure that $ntimes n$ matrix $A$ is invertible. Am I correct?










share|cite|improve this question











$endgroup$








  • 1




    $begingroup$
    @Suraj You should probably give your response in an answer instead of comments
    $endgroup$
    – Shubham Johri
    Dec 19 '18 at 17:04










  • $begingroup$
    so 3 is the only conditions that ensures that n*n matrix A is invertible
    $endgroup$
    – anders
    Dec 19 '18 at 17:04












  • $begingroup$
    @anders Both $(1),(3)$ ensure the invertibility of $A$
    $endgroup$
    – Shubham Johri
    Dec 19 '18 at 17:05










  • $begingroup$
    oh okey. thank you so much
    $endgroup$
    – anders
    Dec 19 '18 at 17:05










  • $begingroup$
    $(1)$ is true, as $det(A^TA)=det(A^T)cdotdet(A)=det(A)^2ne0impliesdet(A)ne0$.
    $endgroup$
    – Shubham Johri
    Dec 19 '18 at 17:06














1












1








1





$begingroup$



Question: Which of the following conditions / properties ensure that the $ntimes n$ matrix $A$ is invertible?



Possible answers:





  1. $A^Tcdot A$ is invertible

  2. $A^T= A$

  3. $A^2+ A + I = 0$


  4. $AB=BA$, for any invertible matrix $B$




My Answer:
My answer is that number $1$ and $4$ are conditions that ensure that $ntimes n$ matrix $A$ is invertible. Am I correct?










share|cite|improve this question











$endgroup$





Question: Which of the following conditions / properties ensure that the $ntimes n$ matrix $A$ is invertible?



Possible answers:





  1. $A^Tcdot A$ is invertible

  2. $A^T= A$

  3. $A^2+ A + I = 0$


  4. $AB=BA$, for any invertible matrix $B$




My Answer:
My answer is that number $1$ and $4$ are conditions that ensure that $ntimes n$ matrix $A$ is invertible. Am I correct?







linear-algebra matrices






share|cite|improve this question















share|cite|improve this question













share|cite|improve this question




share|cite|improve this question








edited Dec 19 '18 at 17:09









Shubham Johri

5,192717




5,192717










asked Dec 19 '18 at 16:57









andersanders

615




615








  • 1




    $begingroup$
    @Suraj You should probably give your response in an answer instead of comments
    $endgroup$
    – Shubham Johri
    Dec 19 '18 at 17:04










  • $begingroup$
    so 3 is the only conditions that ensures that n*n matrix A is invertible
    $endgroup$
    – anders
    Dec 19 '18 at 17:04












  • $begingroup$
    @anders Both $(1),(3)$ ensure the invertibility of $A$
    $endgroup$
    – Shubham Johri
    Dec 19 '18 at 17:05










  • $begingroup$
    oh okey. thank you so much
    $endgroup$
    – anders
    Dec 19 '18 at 17:05










  • $begingroup$
    $(1)$ is true, as $det(A^TA)=det(A^T)cdotdet(A)=det(A)^2ne0impliesdet(A)ne0$.
    $endgroup$
    – Shubham Johri
    Dec 19 '18 at 17:06














  • 1




    $begingroup$
    @Suraj You should probably give your response in an answer instead of comments
    $endgroup$
    – Shubham Johri
    Dec 19 '18 at 17:04










  • $begingroup$
    so 3 is the only conditions that ensures that n*n matrix A is invertible
    $endgroup$
    – anders
    Dec 19 '18 at 17:04












  • $begingroup$
    @anders Both $(1),(3)$ ensure the invertibility of $A$
    $endgroup$
    – Shubham Johri
    Dec 19 '18 at 17:05










  • $begingroup$
    oh okey. thank you so much
    $endgroup$
    – anders
    Dec 19 '18 at 17:05










  • $begingroup$
    $(1)$ is true, as $det(A^TA)=det(A^T)cdotdet(A)=det(A)^2ne0impliesdet(A)ne0$.
    $endgroup$
    – Shubham Johri
    Dec 19 '18 at 17:06








1




1




$begingroup$
@Suraj You should probably give your response in an answer instead of comments
$endgroup$
– Shubham Johri
Dec 19 '18 at 17:04




$begingroup$
@Suraj You should probably give your response in an answer instead of comments
$endgroup$
– Shubham Johri
Dec 19 '18 at 17:04












$begingroup$
so 3 is the only conditions that ensures that n*n matrix A is invertible
$endgroup$
– anders
Dec 19 '18 at 17:04






$begingroup$
so 3 is the only conditions that ensures that n*n matrix A is invertible
$endgroup$
– anders
Dec 19 '18 at 17:04














$begingroup$
@anders Both $(1),(3)$ ensure the invertibility of $A$
$endgroup$
– Shubham Johri
Dec 19 '18 at 17:05




$begingroup$
@anders Both $(1),(3)$ ensure the invertibility of $A$
$endgroup$
– Shubham Johri
Dec 19 '18 at 17:05












$begingroup$
oh okey. thank you so much
$endgroup$
– anders
Dec 19 '18 at 17:05




$begingroup$
oh okey. thank you so much
$endgroup$
– anders
Dec 19 '18 at 17:05












$begingroup$
$(1)$ is true, as $det(A^TA)=det(A^T)cdotdet(A)=det(A)^2ne0impliesdet(A)ne0$.
$endgroup$
– Shubham Johri
Dec 19 '18 at 17:06




$begingroup$
$(1)$ is true, as $det(A^TA)=det(A^T)cdotdet(A)=det(A)^2ne0impliesdet(A)ne0$.
$endgroup$
– Shubham Johri
Dec 19 '18 at 17:06










1 Answer
1






active

oldest

votes


















1












$begingroup$

$1$- It implies $A$ is invertible because if we assume $|A|=m$ then because $A^{T}A$ is invertible then $m^2 neq 0$ and hence $|A|=m neq0$ and Hence $A$ is invertible.



for $2$ and $4$ Consider $A=0$



$3$- consider $Acdot(-A-I)=(-A-I)cdot A=-A^2-A=I$. Hence $A$ is invertible in this case.



Thus only $1$ and $3$ implies $A$ is invertible






share|cite|improve this answer











$endgroup$













  • $begingroup$
    thank you so much for the help
    $endgroup$
    – anders
    Dec 19 '18 at 17:14











Your Answer





StackExchange.ifUsing("editor", function () {
return StackExchange.using("mathjaxEditing", function () {
StackExchange.MarkdownEditor.creationCallbacks.add(function (editor, postfix) {
StackExchange.mathjaxEditing.prepareWmdForMathJax(editor, postfix, [["$", "$"], ["\\(","\\)"]]);
});
});
}, "mathjax-editing");

StackExchange.ready(function() {
var channelOptions = {
tags: "".split(" "),
id: "69"
};
initTagRenderer("".split(" "), "".split(" "), channelOptions);

StackExchange.using("externalEditor", function() {
// Have to fire editor after snippets, if snippets enabled
if (StackExchange.settings.snippets.snippetsEnabled) {
StackExchange.using("snippets", function() {
createEditor();
});
}
else {
createEditor();
}
});

function createEditor() {
StackExchange.prepareEditor({
heartbeatType: 'answer',
autoActivateHeartbeat: false,
convertImagesToLinks: true,
noModals: true,
showLowRepImageUploadWarning: true,
reputationToPostImages: 10,
bindNavPrevention: true,
postfix: "",
imageUploader: {
brandingHtml: "Powered by u003ca class="icon-imgur-white" href="https://imgur.com/"u003eu003c/au003e",
contentPolicyHtml: "User contributions licensed under u003ca href="https://creativecommons.org/licenses/by-sa/3.0/"u003ecc by-sa 3.0 with attribution requiredu003c/au003e u003ca href="https://stackoverflow.com/legal/content-policy"u003e(content policy)u003c/au003e",
allowUrls: true
},
noCode: true, onDemand: true,
discardSelector: ".discard-answer"
,immediatelyShowMarkdownHelp:true
});


}
});














draft saved

draft discarded


















StackExchange.ready(
function () {
StackExchange.openid.initPostLogin('.new-post-login', 'https%3a%2f%2fmath.stackexchange.com%2fquestions%2f3046609%2fwhich-of-the-following-conditions-properties-ensures-that-the-n-times-n-matr%23new-answer', 'question_page');
}
);

Post as a guest















Required, but never shown

























1 Answer
1






active

oldest

votes








1 Answer
1






active

oldest

votes









active

oldest

votes






active

oldest

votes









1












$begingroup$

$1$- It implies $A$ is invertible because if we assume $|A|=m$ then because $A^{T}A$ is invertible then $m^2 neq 0$ and hence $|A|=m neq0$ and Hence $A$ is invertible.



for $2$ and $4$ Consider $A=0$



$3$- consider $Acdot(-A-I)=(-A-I)cdot A=-A^2-A=I$. Hence $A$ is invertible in this case.



Thus only $1$ and $3$ implies $A$ is invertible






share|cite|improve this answer











$endgroup$













  • $begingroup$
    thank you so much for the help
    $endgroup$
    – anders
    Dec 19 '18 at 17:14
















1












$begingroup$

$1$- It implies $A$ is invertible because if we assume $|A|=m$ then because $A^{T}A$ is invertible then $m^2 neq 0$ and hence $|A|=m neq0$ and Hence $A$ is invertible.



for $2$ and $4$ Consider $A=0$



$3$- consider $Acdot(-A-I)=(-A-I)cdot A=-A^2-A=I$. Hence $A$ is invertible in this case.



Thus only $1$ and $3$ implies $A$ is invertible






share|cite|improve this answer











$endgroup$













  • $begingroup$
    thank you so much for the help
    $endgroup$
    – anders
    Dec 19 '18 at 17:14














1












1








1





$begingroup$

$1$- It implies $A$ is invertible because if we assume $|A|=m$ then because $A^{T}A$ is invertible then $m^2 neq 0$ and hence $|A|=m neq0$ and Hence $A$ is invertible.



for $2$ and $4$ Consider $A=0$



$3$- consider $Acdot(-A-I)=(-A-I)cdot A=-A^2-A=I$. Hence $A$ is invertible in this case.



Thus only $1$ and $3$ implies $A$ is invertible






share|cite|improve this answer











$endgroup$



$1$- It implies $A$ is invertible because if we assume $|A|=m$ then because $A^{T}A$ is invertible then $m^2 neq 0$ and hence $|A|=m neq0$ and Hence $A$ is invertible.



for $2$ and $4$ Consider $A=0$



$3$- consider $Acdot(-A-I)=(-A-I)cdot A=-A^2-A=I$. Hence $A$ is invertible in this case.



Thus only $1$ and $3$ implies $A$ is invertible







share|cite|improve this answer














share|cite|improve this answer



share|cite|improve this answer








edited Dec 19 '18 at 17:10









Shubham Johri

5,192717




5,192717










answered Dec 19 '18 at 17:08







user408906



















  • $begingroup$
    thank you so much for the help
    $endgroup$
    – anders
    Dec 19 '18 at 17:14


















  • $begingroup$
    thank you so much for the help
    $endgroup$
    – anders
    Dec 19 '18 at 17:14
















$begingroup$
thank you so much for the help
$endgroup$
– anders
Dec 19 '18 at 17:14




$begingroup$
thank you so much for the help
$endgroup$
– anders
Dec 19 '18 at 17:14


















draft saved

draft discarded




















































Thanks for contributing an answer to Mathematics Stack Exchange!


  • Please be sure to answer the question. Provide details and share your research!

But avoid



  • Asking for help, clarification, or responding to other answers.

  • Making statements based on opinion; back them up with references or personal experience.


Use MathJax to format equations. MathJax reference.


To learn more, see our tips on writing great answers.




draft saved


draft discarded














StackExchange.ready(
function () {
StackExchange.openid.initPostLogin('.new-post-login', 'https%3a%2f%2fmath.stackexchange.com%2fquestions%2f3046609%2fwhich-of-the-following-conditions-properties-ensures-that-the-n-times-n-matr%23new-answer', 'question_page');
}
);

Post as a guest















Required, but never shown





















































Required, but never shown














Required, but never shown












Required, but never shown







Required, but never shown

































Required, but never shown














Required, but never shown












Required, but never shown







Required, but never shown







Popular posts from this blog

Quarter-circle Tiles

build a pushdown automaton that recognizes the reverse language of a given pushdown automaton?

Mont Emei